[seqfan] Re: An arithmetic conjecture

Rainer Rosenthal r.rosenthal at web.de
Tue Mar 17 17:15:27 CET 2009


David Wilson wrote:
> Statistically, you can argue that ...
> The statement
> 
> For all m > 26 there exists some k > 0 such that floor(2^m / 3^k) = 3 (mod 6).
> 
> is similar. It can be restated as ...

Hello David

Thank you so much for your remarkable lines!
It will take me some time to get through. I went through
UPINT yesterday again, but couldn't find anything related to
Peter Luschny's (mod 6) problem. So I am still searching
for a related problem in UPINT (Unsolved Problems in Number
Theory, by R. K. Guy). Any hint is welcome.

Cheers,
Rainer




More information about the SeqFan mailing list